Problematic polynomial

Algebra Level 5

f ( x ) = 4 x 6 + 3 x 4 + 2 x 3 + 7 \large f\left( x \right) =4{ x }^{ 6 }+3{ x }^{ 4 }+2{ x }^{ 3 }+7

If f ( x ) f(x) is polynomial having roots r i { r }_{ i } for i i equal to 1 1 to 6 6 .Then find the value of k = 1 6 ( r i r k ) 4 \displaystyle \sum _{ k=1 }^{ 6 }{ { { \left( \frac { \prod { { r }_{ i } } }{ { r }_{ k } } \right) }^{ 4 } } } . Give your answer to 3 decimal places.

This problem is original.


The answer is -16.078.

This section requires Javascript.
You are seeing this because something didn't load right. We suggest you, (a) try refreshing the page, (b) enabling javascript if it is disabled on your browser and, finally, (c) loading the non-javascript version of this page . We're sorry about the hassle.

3 solutions

Shivamani Patil
Jul 22, 2015

r i = 7 4 \prod { { r }_{ i } } =\frac { 7 }{ 4 } by vieta's formulas.So

k = 1 6 ( r i r k ) 4 = ( 7 4 ) 4 k = 1 6 1 r k 4 \sum _{ k=1 }^{ 6 }{ { { { \left( \frac { \prod { { r }_{ i } } }{ { r }_{ k } } \right) }^{ 4 } } } } ={ \left( \frac { 7 }{ 4 } \right) }^{ 4 }\sum _{ k=1 }^{ 6 }{ \frac { 1 }{ { { r }_{ k } }^{ 4 } } }

Now to find polynomial whose roots are 1 r k \frac { 1 }{ { r }_{ k } } for k = k= 1 1 to 6 6 is obtained by replacing x x with 1 x \frac { 1 }{ x } in f ( x ) f(x) .

f ( 1 x ) = 4 1 x 6 + 3 1 x 4 + 2 1 x 3 + 7 = 7 x 6 + 2 x 3 + 3 x 2 = 0 f\left( \frac { 1 }{ x } \right) =4\frac { 1 }{ { x }^{ 6 } } +3\frac { 1 }{ { x }^{ 4 } } +2\frac { 1 }{ { x }^{ 3 } } +7=7{ x }^{ 6 }+2{ x }^{ 3 }+3{ x }^{ 2 }=0

So f ( 1 x ) f\left( \frac { 1 }{ x } \right) has roots ( 1 r k (\frac { 1 }{ { r }_{ k } } for k = k= 1 1 to 6 6 ).

Now we have to find k = 1 6 1 r k 4 \sum _{ k=1 }^{ 6 }{ \frac { 1 }{ { { r }_{ k } }^{ 4 } } } .

Now we use newton's sums for f ( 1 x ) f\left( \frac { 1 }{ x } \right) .

We have

{ P 1 = 0 P 2 = 0 P 3 = 6 7 P 4 = 12 7 \begin{cases} {{ P }_{ 1 }=0} \\ {{ P }_{ 2 }=0} \\ {{ P }_{ 3 }=\frac { -6 }{ 7 } } \\ { { P }_{ 4 }=\frac { -12 }{ 7 } } \end{cases}

Here P n = i = 1 6 1 ( r i ) n { P }_{ n }=\sum _{ i=1 }^{ 6 }{ \frac { 1 }{ { \left( { r }_{ i } \right) }^{ n } } }

Therefore k = 1 6 1 r k 4 = 12 7 \sum _{ k=1 }^{ 6 }{ \frac { 1 }{ { { r }_{ k } }^{ 4 } } } =\frac { -12 }{ 7 }

Therefore

k = 1 6 ( r i r k ) 4 \sum _{ k=1 }^{ 6 }{ { { { \left( \frac { \prod { { r }_{ i } } }{ { r }_{ k } } \right) }^{ 4 } } } }

= ( 7 4 ) 4 × 12 7 ={ \left( \frac { 7 }{ 4 } \right) }^{ 4 }\times \frac { -12 }{ 7 }

= 16.078 =-16.078

Moderator note:

For clarity, you should show the working for the values of P 1 , P 2 , P 3 , P 4 P_1, P_2, P_3, P_4 .

Aareyan Manzoor
Oct 22, 2015

best way: we get by vietas that ( x 2 ) = 0 2 2 3 4 = 3 2 \sum (x^2)=0^2-2*\frac{3}{4}=-\frac{3}{2} ( 1 x ) = 0 7 4 = 0 \sum (\dfrac{1}{x})=\frac{0}{\frac{7}{4}}=0 ( x ) = 7 4 \prod(x)=\dfrac{7}{4} the given expression becomes 343 256 ( 7 x 4 ) \dfrac{-343}{256}\sum (\dfrac{-7}{x^4}) now, 44 x 6 + 3 x 4 + 2 x 3 + 7 = 0 7 = 4 x 6 + 3 x 4 + 2 x 3 7 x 4 = 4 x 2 + 3 + 2 x ( 7 x 4 ) = 4 ( x 2 ) + 18 + 2 ( 1 x ) = 4 ( 3 2 ) + 18 + 2 0 = 12 \begin{array}{c}44x^6+3x^4+2x^3+7=0\\ -7=4x^6+3x^4+2x^3\\ \dfrac{-7}{x^4}=4x^2+3+\frac{2}{x}\\ \sum (\dfrac{-7}{x^4})=4\sum(x^2)+18+2\sum(\frac{1}{x})=4(-\dfrac{3}{2})+18+2*0=12\end{array} 343 256 ( 7 x 4 ) = 343 256 12 = 16.078 ( 3 d p ) \dfrac{-343}{256}\sum (\dfrac{-7}{x^4})=\dfrac{-343}{256}*12=\boxed{-16.078}(3 dp)

Chew-Seong Cheong
Sep 10, 2015

Let f ( x ) = k = 0 6 a k x k = 4 x 6 + 3 x 4 + 2 x 3 + 7 f(x) = \displaystyle \sum_{k=0}^6 a_kx^k = 4x^6+3x^4+2x^3+7

a 0 = 7 \Rightarrow a_0 = 7 , a 1 = 0 a_1 = 0 , a 2 = 0 a_2 = 0 , a 3 = 2 a_3 = 2 , a 4 = 3 a_4 = 3 , a 5 = 0 a_5 = 0 and a 6 = 4 a_6 = 4 .

k = 1 6 ( r i r k ) 4 = k = 1 6 ( 7 4 r k ) 4 By Vieta’s formulas r i = a 0 a 6 = 7 4 = ( 7 4 ) 4 k = 1 6 1 r k 4 By Newton’s sums method = ( 7 4 ) 4 ( i = 1 6 1 r i k = 1 6 1 r k 3 c y c 1 r i r j k = 1 6 1 r k 2 + c y c 1 r h r i r j k = 1 6 1 r k 4 c y c 1 r g r h r i r j ) = ( 7 4 ) 4 ( r f r g r h r i r j r i k = 1 6 1 r k 3 r g r h r i r j r i k = 1 6 1 r k 2 + r h r i r j r i k = 1 6 1 r k 4 r i r j r i ) = ( 7 4 ) 4 ( a 1 / a 6 a 0 / a 6 k = 1 6 1 r k 3 a 2 / a 6 a 0 / a 6 k = 1 6 1 r k 2 + a 3 / a 6 a 0 / a 6 a 1 / a 6 a 0 / a 6 4 a 4 / a 6 a 0 / a 6 ) = ( 7 4 ) 4 ( a 1 a 0 k = 1 6 1 r k 3 a 2 a 0 k = 1 6 1 r k 2 + a 3 a 0 × a 1 a 0 4 × a 4 a 0 ) = ( 7 4 ) 4 [ 0 4 k = 1 6 1 r k 3 0 4 k = 1 6 1 r k 2 + 2 4 ( 0 4 ) 4 ( 3 4 ) ] = ( 7 4 ) 4 [ 4 ( 3 4 ) ] = 16.078125 \begin{aligned} \sum_{k=1}^6 \left(\frac{\color{#3D99F6}{\prod r_i}}{r_k} \right)^4 & = \sum_{k=1}^6 \left(\frac{\color{#3D99F6}{ \frac{7}{4}}}{r_k} \right)^4 \quad \quad \quad \small \color{#3D99F6}{\text{By Vieta's formulas } \prod r_i = \frac{a_0}{a_6} = \frac{7}{4}} \\ & = \left(\frac{7}{4}\right)^4 \sum_{k=1}^6 \frac{1}{r_k^4} \quad \quad \small \color{#3D99F6}{\text{By Newton's sums method}} \\ & = \left(\frac{7}{4}\right)^4 \left( \color{#D61F06}{\sum_{i=1}^6 \frac{1}{r_i}} \sum_{k=1}^6 \frac{1}{r_k^3} - \sum_{cyc} \frac{1}{r_ir_j} \sum_{k=1}^6 \frac{1}{r_k^2} + \sum_{cyc} \frac{1}{r_hr_ir_j} \color{#D61F06}{\sum_{k=1}^6 \frac{1}{r_k}} - 4\sum_{cyc} \frac{1}{r_gr_hr_ir_j} \right) \\ & = \left(\frac{7}{4}\right)^4 \left(\color{#D61F06}{\frac{\sum r_fr_gr_hr_ir_j}{\prod r_i}} \sum_{k=1}^6 \frac{1}{r_k^3} - \frac{\sum r_gr_hr_ir_j}{\prod r_i} \sum_{k=1}^6 \frac{1}{r_k^2} + \frac{\sum r_hr_ir_j}{\prod r_i} \color{#D61F06}{\sum_{k=1}^6 \frac{1}{r_k}} - 4\frac{\sum r_ir_j}{\prod r_i} \right) \\ & = \left(\frac{7}{4}\right)^4 \left(\color{#D61F06}{\frac{a_1/a_6}{a_0/a_6}} \sum_{k=1}^6 \frac{1}{r_k^3} - \frac{a_2/a_6}{a_0/a_6} \sum_{k=1}^6 \frac{1}{r_k^2} + \frac{a_3/a_6}{a_0/a_6} \color{#D61F06}{\frac{a_1/a_6}{a_0/a_6}} - 4 \frac{a_4/a_6}{a_0/a_6} \right) \\ & = \left(\frac{7}{4}\right)^4 \left(\frac{a_1}{a_0} \sum_{k=1}^6 \frac{1}{r_k^3} - \frac{a_2}{a_0} \sum_{k=1}^6 \frac{1}{r_k^2} + \frac{a_3}{a_0} \times \frac{a_1}{a_0} - 4\times \frac{a_4}{a_0} \right) \\ & = \left(\frac{7}{4}\right)^4 \left[\frac{0}{4} \sum_{k=1}^6 \frac{1}{r_k^3} - \frac{0}{4} \sum_{k=1}^6 \frac{1}{r_k^2} + \frac{-2}{4}\left(\frac{0}{4}\right) - 4 \left(\frac{3}{4} \right) \right] \\ & = \left(\frac{7}{4}\right)^4 \left[- 4 \left(\frac{3}{4} \right)\right] \\ & = \boxed{-16.078125} \end{aligned}

0 pending reports

×

Problem Loading...

Note Loading...

Set Loading...